¿Cómo derivar la corriente de Goldstone-Wilczek?

Estoy leyendo el artículo de este célebre Goldstone-Wilczek sobre el número cuántico fraccionario. En particular, derivaron para el siguiente Dirac Lagrangiano ( ϕ 1 y ϕ 2 son campos escalares)

L = i ψ ¯ γ m m ψ + gramo ψ ¯ ( ϕ 1 + i γ 5 ϕ 2 ) ψ

que el valor esperado de las lecturas actuales ( θ = broncearse 1 ( ϕ 2 / ϕ 1 ) )

j m = 1 2 π ϵ m v v θ
,

que es la ecuación (2). Es posible entenderlo vía reducción dimensional o bosonización. Pero me gustaría entenderlo desde una forma teórica de campo. En particular tengo dos preguntas:

  1. ¿Cómo obtener el diagrama de Feynman en la Fig.3? (Soy consciente de la pregunta similar aquí, pero no responde a mi pregunta, y el diagrama de Feynman a continuación es de ese hilo) (Para mayor claridad, la línea curva es actual, la línea sólida es fermión y la línea discontinua es escalar)
  2. ¿Cómo calcular este diagrama de Feynman?la línea curva es actual, la línea continua es fermión y la línea discontinua es escalar

He buscado en Internet y bastantes referencias y no pude resolverlo. Cualquier ayuda es apreciada.

Puede o no optar por la deconstrucción quiral como en Hill & Zachos 2005 , sección II.
Consulte este artículo, arxiv.org/abs/1306.0998 y las notas de la conferencia "Introducción a la bosonización" de D. Sénéchal. Creo que también ayudará.

Respuestas (2)

Es interesante ver su pregunta anterior no solo a la luz del enfoque de Goldstone-Wilczek (GW) (GW ha proporcionado un método para calcular la carga de fermiones inducida por un perfil clásico), sino también calculando 1 / 2 -Carga de fermiones encontrada por Jackiw-Rebbi usando el método GW. Para simplificar, consideremos el caso 1+1D, y consideremos el Z 2 muro de dominio y el 1 / 2 -cargo encontrado por Jackiw-Rebbi. La construcción, válida para sistemas 1+1D, funciona de la siguiente manera.

Considere un Lagrangiano que describe fermiones sin espín ψ ( X ) acoplado a un perfil de fondo clásico λ ( X ) a través de un término λ ψ σ 3 ψ . En la fase de alta temperatura, el vev de λ es cero y no se genera masa para los fermiones. En la fase de baja temperatura, el λ adquiere dos valores de vacío degenerados ± λ que están relacionados por un Z 2 simetría. Genéricamente tenemos

λ porque ( ϕ ( X ) θ ( X ) ) ,
donde usamos el diccionario de bosonización ψ σ 3 ψ porque ( ϕ ( X ) ) y un cambio de fase Δ θ = π captura la existencia de una pared de dominio que separa regiones con valores opuestos de la vev de λ . Por el hecho de que la densidad de fermiones
ρ ( X ) = ψ ( X ) ψ ( X ) = 1 2 π X ϕ ( X ) ,
y la corriente
j m = ψ γ m ψ = 1 2 π ϵ m v v ϕ ,
se sigue que la carga inducida q dw en la torcedura por un muro de dominio es
q dw = X 0 ε X 0 + ε d X ρ ( X ) = X 0 ε X 0 + ε d X 1 2 π X ϕ ( X ) = 1 2 π π = 1 2 ,
dónde X 0 denota el centro de la pared del dominio.

Puede intentar extenderse a otras dimensiones, pero es posible que deba tener cuidado y es posible que no pueda usar la bosonización.

Ver más detalles de la derivación aquí en la página 13 de este trabajo .

Para derivar esto, necesita hacer una transformación quiral local al fermión para hacer el término de dos masas ( ϕ 1 + γ 5 ϕ 2 ) un término de masa de fermión estándar, y esta transformación quiral local provocará un término de interacción de la energía cinética.

Hola Feng, si desea utilizar MathJax, debe encerrar sus expresiones en $etiquetas (para el modo en línea) o $$etiquetas (para el modo de visualización).
para agregar a lo que dijo @ J.Murray, la "barra inclinada" correcta es la barra invertida (), no la barra inclinada (/). Mientras tanto, editaré su respuesta por usted, pero téngala en cuenta para futuras respuestas/preguntas.